Difference between revisions of "1989 AIME Problems/Problem 9"

m (Solution 2: LaTeX the solution.)
m (Solution 2: Correcting errors is commendable, but one-character revision is too small for credits, right? PM me if you disagree. Sorry.)
(16 intermediate revisions by 2 users not shown)
Line 2: Line 2:
 
One of Euler's conjectures was disproved in the 1960s by three American mathematicians when they showed there was a positive integer such that <cmath>133^5+110^5+84^5+27^5=n^{5}.</cmath> Find the value of <math>n</math>.
 
One of Euler's conjectures was disproved in the 1960s by three American mathematicians when they showed there was a positive integer such that <cmath>133^5+110^5+84^5+27^5=n^{5}.</cmath> Find the value of <math>n</math>.
  
== Solution 1 ==
+
== Solution 1 (FLT, CRT, Inequalities) ==
 +
Taking the given equation modulo <math>2,3,</math> and <math>5,</math> respectively, we have
 +
<cmath>\begin{align*}
 +
n^5&\equiv0\pmod{2}, \\
 +
n^5&\equiv0\pmod{3}, \\
 +
n^5&\equiv4\pmod{5}.
 +
\end{align*}</cmath>
 +
By either Fermat's Little Theorem (FLT) or inspection, we get
 +
<cmath>\begin{align*}
 +
n&\equiv0\pmod{2}, \\
 +
n&\equiv0\pmod{3}, \\
 +
n&\equiv4\pmod{5}.
 +
\end{align*}</cmath>
 +
By either the Chinese Remainder Theorem (CRT) or inspection, we get <math>n\equiv24\pmod{30}.</math>
 +
 
 +
It is clear that <math>n>133,</math> so the possible values for <math>n</math> are <math>144,174,204,\ldots.</math> Note that
 +
<cmath>\begin{align*}
 +
n^5&=133^5+110^5+84^5+27^5 \\
 +
&<133^5+110^5+(84+27)^5 \\
 +
&=133^5+110^5+111^5 \\
 +
&<3\cdot133^5,
 +
\end{align*}</cmath>
 +
from which <math>\left(\frac{n}{133}\right)^5<3.</math>
 +
 
 +
If <math>n\geq174,</math> then
 +
<cmath>\begin{align*}
 +
\left(\frac{n}{133}\right)^5&>1.3^5 \\
 +
&=1.3^2\cdot1.3^2\cdot1.3 \\
 +
&>1.6\cdot1.6\cdot1.3 \\
 +
&=2.56\cdot1.3 \\
 +
&>2.5\cdot1.2 \\
 +
&=3,
 +
\end{align*}</cmath>
 +
which arrives at a contradiction. Therefore, we conclude that <math>n=\boxed{144}.</math>
 +
 
 +
~MRENTHUSIASM
 +
 
 +
== Solution 2 ==
 
Note that <math>n</math> is even, since the LHS consists of two odd and two even numbers. By Fermat's Little Theorem, we know <math>n^5\equiv n\pmod{5}.</math> Hence, <cmath>n\equiv3+0+4+2\equiv4\pmod{5}.</cmath>
 
Note that <math>n</math> is even, since the LHS consists of two odd and two even numbers. By Fermat's Little Theorem, we know <math>n^5\equiv n\pmod{5}.</math> Hence, <cmath>n\equiv3+0+4+2\equiv4\pmod{5}.</cmath>
Continuing, we examine the equation modulo <math>3,</math> <cmath>n\equiv1-1+0+0\equiv0\pmod{5}.</cmath>
+
Continuing, we examine the equation modulo <math>3,</math> <cmath>n\equiv1-1+0+0\equiv0\pmod{3}.</cmath>
Thus, <math>n</math> is divisible by three and leaves a remainder of four when divided by <math>5.</math> It's obvious that <math>n>133,</math> so the only possibilities are <math>n = 144</math> or <math>n \geq 174.</math> It quickly becomes apparent that <math>174</math> is much too large, so <math>n</math> must be <math>\boxed{144}.</math>
+
Thus, <math>n</math> is divisible by three and leaves a remainder of four when divided by <math>5.</math> It is obvious that <math>n>133,</math> so the only possibilities are <math>n = 144</math> or <math>n \geq 174.</math> It quickly becomes apparent that <math>174</math> is much too large, so <math>n</math> must be <math>\boxed{144}.</math>
  
 
~Azjps (Solution)
 
~Azjps (Solution)
Line 11: Line 48:
 
~MRENTHUSIASM (Reformatting)
 
~MRENTHUSIASM (Reformatting)
  
== Solution 2 ==
+
== Solution 3 ==
We can cheat a little bit and approximate, since we are dealing with such large numbers. As above, <math>n^5\equiv n\pmod{5}</math>, and it is easy to see that <math>n^5\equiv n\pmod 2</math>. Therefore, <math>133^5+110^5+84^5+27^5\equiv 3+0+4+7\equiv 4\pmod{10}</math>, so the last digit of <math>n</math> is <math>4</math>.
+
We can cheat a little bit and approximate, since we are dealing with such large numbers. As above, <math>n^5\equiv n\pmod{5},</math> and it is easy to see that <math>n^5\equiv n\pmod 2.</math> Therefore, <math>133^5+110^5+84^5+27^5\equiv 3+0+4+7\equiv 4\pmod{10},</math> so the last digit of <math>n</math> is <math>4.</math>
 
 
We notice that <math>133,110,84,</math> and <math>27</math> are all very close or equal to multiples of <math>27</math>. We can rewrite <math>n^5</math> as approximately equal to <math>27^5(5^5+4^5+3^5+1^5) = 27^5(4393)</math>. This means <math>\frac{n}{27}</math> must be close to <math>4393</math>.
 
  
Note that <math>134</math> will obviously be too small, so we try <math>144</math>: <cmath>\left(\frac{144}{27}\right)^5=\left(\frac{16}{3}\right)^5.</cmath> Bashing through the division, we find that <math>\frac{1048576}{243}\approx 4315</math>, which is very close to <math>4393</math>. It is clear that <math>154</math> will not give any closer of an answer, given the rate that fifth powers grow, so we can safely assume that <math>\boxed{144}</math> is the answer.
+
We notice that <math>133,110,84,</math> and <math>27</math> are all very close or equal to multiples of <math>27.</math> We can rewrite <math>n^5</math> as approximately equal to <math>27^5(5^5+4^5+3^5+1^5) = 27^5(4393).</math> This means <math>\frac{n}{27}</math> must be close to <math>4393.</math>
  
==Solution 3==
+
Note that <math>134</math> will obviously be too small, so we try <math>144</math> and get <math>\left(\frac{144}{27}\right)^5=\left(\frac{16}{3}\right)^5.</math> Bashing through the division, we find that <math>\frac{1048576}{243}\approx 4315,</math> which is very close to <math>4393.</math> It is clear that <math>154</math> will not give any closer of an answer, given the rate that fifth powers grow, so we can safely assume that <math>\boxed{144}</math> is the answer.
In this solution we take advantage of the large numbers and utilize parity properties to give us a very good guess at the answer. The units digit of each of the four numbers is 3, 0, 4, and 7, respectively. This means the units digit of <math>n^5</math> is 4. This tells us <math>n</math> is even. Since we are dealing with enormous numbers, <math>n</math> should not be that far from 133. <math>n</math>'s unit digit is 0, 2, 4, 6, or 8. When to the power of 5 they each give 0, 2, 4, 6, and 8 as unit digits. This further clues us that <math>n</math> ends in 4.  
 
  
<math>n</math> is obviously larger than 133, so we start with 134. Now we need a way of distinguishing between numbers with unit digit 4. This can be done by simply solving up to the hundreds digit of <math>133^5</math>, <math>110^5</math>, <math>84^5</math>, and <math>27^5</math>, which isn't that difficult. For 133, all that has to be done is square it and take the last three digits, 689, and raise them to the power of 2 again, 721, then multiply this by 133. This gives us 893. Doing this for each tells us <math>n^5</math> ends in 224. Testing 134 the same way we did with 133 gives us 424, 144 gives us 224, 154 gives us 024, 164 gives us 824, 624, 424, and finally 194 also gives 224.
+
==Solution 4==
 +
In this solution we take advantage of the large numbers and utilize parity properties to give us a very good guess at the answer. The units digits of <math>133^5, 110^5, 84^5, 27^5</math> are <math>3, 0, 4, 7,</math> respectively, so the units digit of <math>n^5</math> is <math>4.</math> This tells us <math>n</math> is even. Since we are dealing with enormous numbers, <math>n</math> should not be that far from <math>133.</math> Note that <math>n</math>'s units digit is <math>0, 2, 4, 6,</math> or <math>8.</math> When to the power of <math>5,</math> they each give <math>0, 2, 4, 6,</math> and <math>8</math> as the units digits. This further clues us that <math>n</math> ends in <math>4.</math>
  
By simply using our judgement it's hard to imagine the sum of those powers of 5 to be <math>194^5</math>. The answer is <math>\boxed{144}</math>.
+
Clearly, <math>n>133,</math> so we start with <math>134.</math> Now we need a way of distinguishing between numbers with units digit <math>4.</math> We can do this by finding the last three digits for each of <math>133^5, 110^5, 84^5,</math> and <math>27^5,</math> which is not that difficult. For <math>133^5,</math> we have
 +
<cmath>\begin{align*}
 +
133^5&=133^2\cdot133^2\cdot133 \\
 +
&\equiv689\cdot689\cdot133 \\
 +
&\equiv721\cdot133 \\
 +
&\equiv893\pmod{1000}.
 +
\end{align*}</cmath>
 +
By the same reasoning, we get
 +
<cmath>\begin{align*}
 +
n^5&=133^5+110^5+84^5+27^5 \\
 +
&\equiv893+0+424+907 \\
 +
&\equiv224\pmod{1000}.
 +
\end{align*}</cmath>
 +
Note that
 +
<cmath>\begin{align*}
 +
134^5&\equiv424\pmod{1000}, \\
 +
144^5&\equiv224\pmod{1000}, \\
 +
154^5&\equiv024\pmod{1000}, \\
 +
164^5&\equiv824\pmod{1000}, \\
 +
174^5&\equiv624\pmod{1000}, \\
 +
184^5&\equiv424\pmod{1000}, \\
 +
194^5&\equiv224\pmod{1000}.
 +
\end{align*}</cmath>
 +
By observations, <math>n=194</math> is obviously an overestimate. So, the answer is <math>n=\boxed{144}.</math>
  
 +
~jackshi2006 (Solution)
  
-jackshi2006
+
~MRENTHUSIASM (Revisions and <math>\LaTeX</math> Adjustments)
  
 
== See also ==
 
== See also ==

Revision as of 06:31, 10 September 2021

Problem

One of Euler's conjectures was disproved in the 1960s by three American mathematicians when they showed there was a positive integer such that \[133^5+110^5+84^5+27^5=n^{5}.\] Find the value of $n$.

Solution 1 (FLT, CRT, Inequalities)

Taking the given equation modulo $2,3,$ and $5,$ respectively, we have \begin{align*} n^5&\equiv0\pmod{2}, \\ n^5&\equiv0\pmod{3}, \\ n^5&\equiv4\pmod{5}. \end{align*} By either Fermat's Little Theorem (FLT) or inspection, we get \begin{align*} n&\equiv0\pmod{2}, \\ n&\equiv0\pmod{3}, \\ n&\equiv4\pmod{5}. \end{align*} By either the Chinese Remainder Theorem (CRT) or inspection, we get $n\equiv24\pmod{30}.$

It is clear that $n>133,$ so the possible values for $n$ are $144,174,204,\ldots.$ Note that \begin{align*} n^5&=133^5+110^5+84^5+27^5 \\ &<133^5+110^5+(84+27)^5 \\ &=133^5+110^5+111^5 \\ &<3\cdot133^5, \end{align*} from which $\left(\frac{n}{133}\right)^5<3.$

If $n\geq174,$ then \begin{align*} \left(\frac{n}{133}\right)^5&>1.3^5 \\ &=1.3^2\cdot1.3^2\cdot1.3 \\ &>1.6\cdot1.6\cdot1.3 \\ &=2.56\cdot1.3 \\ &>2.5\cdot1.2 \\ &=3, \end{align*} which arrives at a contradiction. Therefore, we conclude that $n=\boxed{144}.$

~MRENTHUSIASM

Solution 2

Note that $n$ is even, since the LHS consists of two odd and two even numbers. By Fermat's Little Theorem, we know $n^5\equiv n\pmod{5}.$ Hence, \[n\equiv3+0+4+2\equiv4\pmod{5}.\] Continuing, we examine the equation modulo $3,$ \[n\equiv1-1+0+0\equiv0\pmod{3}.\] Thus, $n$ is divisible by three and leaves a remainder of four when divided by $5.$ It is obvious that $n>133,$ so the only possibilities are $n = 144$ or $n \geq 174.$ It quickly becomes apparent that $174$ is much too large, so $n$ must be $\boxed{144}.$

~Azjps (Solution)

~MRENTHUSIASM (Reformatting)

Solution 3

We can cheat a little bit and approximate, since we are dealing with such large numbers. As above, $n^5\equiv n\pmod{5},$ and it is easy to see that $n^5\equiv n\pmod 2.$ Therefore, $133^5+110^5+84^5+27^5\equiv 3+0+4+7\equiv 4\pmod{10},$ so the last digit of $n$ is $4.$

We notice that $133,110,84,$ and $27$ are all very close or equal to multiples of $27.$ We can rewrite $n^5$ as approximately equal to $27^5(5^5+4^5+3^5+1^5) = 27^5(4393).$ This means $\frac{n}{27}$ must be close to $4393.$

Note that $134$ will obviously be too small, so we try $144$ and get $\left(\frac{144}{27}\right)^5=\left(\frac{16}{3}\right)^5.$ Bashing through the division, we find that $\frac{1048576}{243}\approx 4315,$ which is very close to $4393.$ It is clear that $154$ will not give any closer of an answer, given the rate that fifth powers grow, so we can safely assume that $\boxed{144}$ is the answer.

Solution 4

In this solution we take advantage of the large numbers and utilize parity properties to give us a very good guess at the answer. The units digits of $133^5, 110^5, 84^5, 27^5$ are $3, 0, 4, 7,$ respectively, so the units digit of $n^5$ is $4.$ This tells us $n$ is even. Since we are dealing with enormous numbers, $n$ should not be that far from $133.$ Note that $n$'s units digit is $0, 2, 4, 6,$ or $8.$ When to the power of $5,$ they each give $0, 2, 4, 6,$ and $8$ as the units digits. This further clues us that $n$ ends in $4.$

Clearly, $n>133,$ so we start with $134.$ Now we need a way of distinguishing between numbers with units digit $4.$ We can do this by finding the last three digits for each of $133^5, 110^5, 84^5,$ and $27^5,$ which is not that difficult. For $133^5,$ we have \begin{align*} 133^5&=133^2\cdot133^2\cdot133 \\ &\equiv689\cdot689\cdot133 \\ &\equiv721\cdot133 \\ &\equiv893\pmod{1000}. \end{align*} By the same reasoning, we get \begin{align*} n^5&=133^5+110^5+84^5+27^5 \\ &\equiv893+0+424+907 \\ &\equiv224\pmod{1000}. \end{align*} Note that \begin{align*} 134^5&\equiv424\pmod{1000}, \\ 144^5&\equiv224\pmod{1000}, \\ 154^5&\equiv024\pmod{1000}, \\ 164^5&\equiv824\pmod{1000}, \\ 174^5&\equiv624\pmod{1000}, \\ 184^5&\equiv424\pmod{1000}, \\ 194^5&\equiv224\pmod{1000}. \end{align*} By observations, $n=194$ is obviously an overestimate. So, the answer is $n=\boxed{144}.$

~jackshi2006 (Solution)

~MRENTHUSIASM (Revisions and $\LaTeX$ Adjustments)

See also

1989 AIME (ProblemsAnswer KeyResources)
Preceded by
Problem 8
Followed by
Problem 10
1 2 3 4 5 6 7 8 9 10 11 12 13 14 15
All AIME Problems and Solutions

The problems on this page are copyrighted by the Mathematical Association of America's American Mathematics Competitions. AMC logo.png